Difference between revisions of "2017 AMC 12B Problems/Problem 19"

m (Solution 2)
m (Solution 2)
Line 18: Line 18:
 
==Solution 2==
 
==Solution 2==
  
We know that this number is divisible by <math>9</math> because the sum of the digits is <math>270</math>, which is divisible by <math>9</math>. Subtracting 9 from the integer we get <math>1234 \cdots 4335</math>, which is also divisible by <math>5</math>, making it also divisible by <math>45</math>. Thus the remainder is <math>9</math>, or <math>\boxed{\textbf{B}}</math>.
+
We know that this number is divisible by <math>9</math> because the sum of the digits is <math>270</math>, which is divisible by <math>9</math>. If we subtracted <math>9</math> from the integer we would get <math>1234 \cdots 4335</math>, which is also divisible by <math>5</math> and by <math>45</math>. Thus the remainder is <math>9</math>, or <math>\boxed{\textbf{B}}</math>.
  
 
==See Also==
 
==See Also==

Revision as of 16:07, 11 February 2019

Problem

Let $N=123456789101112\dots4344$ be the $79$-digit number that is formed by writing the integers from $1$ to $44$ in order, one after the other. What is the remainder when $N$ is divided by $45$?

$\textbf{(A)}\ 1\qquad\textbf{(B)}\ 4\qquad\textbf{(C)}\ 9\qquad\textbf{(D)}\ 18\qquad\textbf{(E)}\ 44$

Solution

We will consider this number $\bmod\ 5$ and $\bmod\ 9$. By looking at the last digit, it is obvious that the number is $\equiv 4\bmod\ 5$. To calculate the number $\bmod\ 9$, note that

\[123456\cdots 4344 \equiv 1+2+3+4+5+6+7+8+9+(1+0)+(1+1)+\cdots+(4+3)+(4+4) \equiv 1+2+\cdots+44 \bmod\ 9,\]

so it is equivalent to

\[\frac{44\cdot 45}{2} = 22\cdot 45 \equiv 0\bmod\ 9.\]

Let $x$ be the remainder when this number is divided by $45$. We know that $x\equiv 0 \pmod {9}$ and $x\equiv 4 \pmod {5}$, so by the Chinese remainder theorem, since $9(-1)\equiv 1 \pmod{5}$, $x\equiv 5(0)+9(-1)(4) \pmod {5\cdot 9}$, or $x\equiv -36 \equiv 9 \pmod {45}$. So the answer is $\boxed {\textbf {(C)}}$.

Solution 2

We know that this number is divisible by $9$ because the sum of the digits is $270$, which is divisible by $9$. If we subtracted $9$ from the integer we would get $1234 \cdots 4335$, which is also divisible by $5$ and by $45$. Thus the remainder is $9$, or $\boxed{\textbf{B}}$.

See Also

2017 AMC 12B (ProblemsAnswer KeyResources)
Preceded by
Problem 18
Followed by
Problem 20
1 2 3 4 5 6 7 8 9 10 11 12 13 14 15 16 17 18 19 20 21 22 23 24 25
All AMC 12 Problems and Solutions
2017 AMC 10B (ProblemsAnswer KeyResources)
Preceded by
Problem 22
Followed by
Problem 24
1 2 3 4 5 6 7 8 9 10 11 12 13 14 15 16 17 18 19 20 21 22 23 24 25
All AMC 10 Problems and Solutions

The problems on this page are copyrighted by the Mathematical Association of America's American Mathematics Competitions. AMC logo.png